Está en la página 1de 5

MATH321 HOMEWORK SOLUTIONS

HOMEWORK #1
Section 1.2: Problems 1, 2, 11, 12, 13, 15, 16, 17, 18
Section 1.3: Problems 1, 8, 10, 13, 17, 19, 24, 25
Section 1.4: Problems 1, 2(a)(d), 3(a), 4(a), 6, 10
Krzysztof Galicki

Problem 1.2.1 (See Answers to Selected Exercises).


Problem 1.2.2

0 0
~0 = O = 0 0
0 0

0
0
0

0
0.
0

Problem 1.2.11 The axioms VS.1,2,5,6,7,8 are trivially satisfied as there is only one
element in V . By definition 0 is the zero vector and it is its own inverse so that (VS.3-4)
are likewise true.
Problem 1.2.12 We know that F(R, R) is a vector space. The zero function f (x) 0
is even. Adding two even functions gives an even function. Finally, if f (x) is even so
is cf (x). Hence, VS.1-8 must all be satisfied (as they are already satisfied for F(R, R)).
That is, the set of even functions in F(R, R) forms a vector subspace.
Problem 1.2.13 The axioms VS.1-3 are satisfied with the only possible zero vector
being (0, 1). The axiom VS.4 is not satisfied as (a1 , 0) has no inverse, i.e., there is no
element (b1 , b2 ) such that
(b1 , b2 ) + (a1 , 0) = (0, 1).
The axioms VS.5-7 are satisfied but VS.8 fails as, for example,
(1 + 1)(a1 , a2 ) = 2(a1 , a2 ) = (2a1 , a2 )
but
(a1 , a2 ) + (a1 , a2 ) = (2a1 , a2 a2 ).

Problem 1.2.15 No. The operation of vector addition is not affected by changing F from
R to C. But scalar multiplication is not really defined on V as, for example, multiplication
by i turns real vectors into vectors with imaginary complex coordinates.
Problem 1.2.16 Yes. The field Q R is a subfield with the same unit. The axioms VS.18 are all satisfied. However, (Mmn (R), R, +, ) and (Mmn (R), Q, +, ) do not have the
same dimension (As we shall see later). The three vector spaces (Mmn (R), R, +, ),
(Mmn (R), Q, +, ) and (Mmn (Q), Q, +, ) are all different.
Problem 1.2.17 No. Axioms VS.1-4 are satisfied. Axiom VS.5 is not satisfied. Axioms
VS.6-7 are fine. Axiom VS.8 fails.
Problem 1.2.18 No. Even axiom VS.1 (commutativity) is not satisfied:
(1, 1) + (2, 2) = (5, 7),

(2, 2) + (1, 1) = (4, 5).

Problem 1.3.1 (See Answers to Selected Exercises).


Problem 1.3.8 In each case we check if (0, 0, 0) is in W R3 first. This holds for (a),
(c), (d), (f). Hence, W2 and W5 are not vector subspaces. Then we check if addition
and scalar multiplication are closed. It is clearly so for W1 , W3 , W4 and these three are
therefore vector spaces. In (f) the quadratic equation ishomogeneous.
If (a1 , a2 , a3 ) is
a solution so is its multiple c(a1 , a2 , a3 ). However, (0, 2, 1) and (0, 2, 1) are both
solutions but the sum

(0, 2, 1) + (0, 2, 1) = (0, 0, 2)


is not. Hence, W6 is not a vector subspace.
Problem 1.3.10 W1 contains the zero vector (0, . . . , 0). The vector addition and scalar
multiplication preserve W1 . W2 does not contain the zero vector.
Problem 1.3.13 The set of functions vanishing at a given point s0 S is a subset of all
functions. The zero function f (s) = 0 belongs to this set. Adding two functions vanishing
at s0 gives a function which also vanishes at s0 . Multiplying a function vanishing at s0
by any element c F gives a function that vanishes at s0 . Hence, this subset is a vector
subspace.
Problem 1.3.17 We use Theorem 1.3 which gives an equivalent definition of a vector
subspace. If (a-c) are satisfied then by (a) W V is non-empty and by (b) x, y W
x + y W , and by (c) x W c F cx W .

Conversely. Suppose W is non-empty and satisfies (b)+(c). Let x W . Using (c) we


have 0 x = ~0 so that ~0 W .
Problem 1.3.19 Let W1 , W2 be vector subspaces of V . Note that if W1 W2 then
W1 W2 = W2 and if W2 W1 then W1 W2 = W1 . Hence, if any inclusion holds
the statement follows. This proves the if statement. To establish the only if part we
assume that neither inclusion holds. That is there exists vectors x in W1 which is not in
W2 and y which is in W2 but not in W1 .
x W1 ,

x 6 W2 ,

y W2 ,

x 6 W1 .

Consider x + y. This is, however, not in W1 W2 . For if x + y W1 W2 then either


x + y = w1

for some w1 W1

x + y = w2

for some w2 W2 .

or
In the first case we get y = w1 x W1 which is a contradiction. In the second case we
get x = w2 y W2 which is a contradiction.
Problem 1.3.24 Clearly W1 W2 = {(0, . . . , 0)} and any vector in F n can be written
(uniquely) as sum of a vector in W1 and W2 . Hence, F n = W1 W2 .
Problem 1.3.25 W1 P (F ) is the set of odd polynomials f (t) = f (t). Likewise,
W2 P (F ) is the set of even polynomials f (t) = f (t). Clearly, no polynomial can be
both even and odd unless f (t) = 0. Any polynomial f (t) can be written
p(t) =

n
X

ai ti = (a0 + a2 t2 + ) + (a1 + a3 t3 + ) = pe (t) + po (t).

i=0

This shows that P (F ) = W1 W2 .

Problem 1.4.1 (See Answers to Selected Exercises).


Problem 1.4.2
(a)


2 2 3 0 2
1
3 3 2 5 7 ' 0

1 1 2 1 3
0

1
0
0

2
1
4

1 3
2 4 '
8 16



1 1 2 0 1
1

0 0

1 2 4
' 0
0 0
0 0 0
0


1 0 3 5
0 1 2 4 .
0 0 0 0

The last matrix is in the RREF. The solution is obtained by setting x2 = t, x4 = s


and

(x1 , x2 , x3 , x4 ) = (t 3s + 5, t, 2s + 4, s) = t(1, 1, 0, 0) + s(3, 0, 2, 1) + (5, 0, 4, 0).


(d)

1 2
1 0
1 1


1
2 0 2

8 5 6 ' 0
0
5 5 3

1
0
0


1 2
2 2 0 2

2 6 5 8 ' 0 1
0 0
1 3 5 1

0
1
0

2
3
0

0 2
5 1 '
5 10

8 0 16
3 0 9 .
0 1 2

The last matrix is in the RREF. The solution is obtained by setting x3 = t. Hence,
(x1 , x2 , x3 , x4 ) = (8t 16, 3t + 9, t, 2) = t(8, 3, 1, 0) + (16, 9, 0, 2).

Problem 1.4.3
(a) Yes. We have
(2, 0, 3) = a(1, 3, 0) + b(2, 4, 1).
This gives immediately b = 3 (last component). As 3a + 4b = 0 (second component)
then a = 4. Now, it remain to check that for (a, b) = (4, 3) we have a + 2b = 2
(first coordinate) which is indeed the case.
Problem 1.4.4
(a) Yes. We must have
x3 3x + 5 = a(x3 + 2x2 x + 1) + b(x3 + 3x2 1),
which gives
a + b = 1, 2a + 3b = 0, a = 3, a b = 5.

All these hold for a = 3, b = 2.


Problem 1.4.6 We have
a(1, 1, 0) + b(1, 0, 1) + c(0, 1, 1) = (a + b, a + c, b + c).
We must show that the equation (x, y, z) = (a + b, a + c, b + c) has a solution for any
(x, y, z). Clearly, this is so, as
b = (x + z y)/2, a = (x + y z)/2, c = (y + z x)/2.

Problem 1.4.10 Let

M1 =

0 0
0 1

, M2 =

1 0
0 0

Then

, M3 =

aM1 + bM2 + cM3 =

b
c

c
a

0 1
1 0

so that span{M1 , M2 , M3 } is the set of all symmetric 2 2 matrices.

También podría gustarte